Matematik

Intergralregning

17. september 2018 af rikke0227 - Niveau: A-niveau

Hej er der nogle der kan hjælpe mig, sidder lidt fast 

Vedhæftet fil: opgave.PNG

Brugbart svar (0)

Svar #1
17. september 2018 af guuoo2


I sidste linje glemmer du at bruge nye grænser,
og du indsætter dx = du selvom du rigtigt har skrevet du = -dx.


Svar #2
17. september 2018 af rikke0227

Jeg forstår ikke helt hvad du mener


Svar #3
17. september 2018 af rikke0227

Det jeg har problemer med er hvordan man intergrer det sidste hvor jeg også har sat spørgsmåltegn. Er usikker fordi at det står i tredje


Brugbart svar (0)

Svar #4
17. september 2018 af guuoo2

Prøv at differentiere  u4 mht. u


Svar #5
17. september 2018 af rikke0227

vil de så blive 3u^2 ? og hvad med tallet 4 ?


Brugbart svar (0)

Svar #6
17. september 2018 af ringstedLC

\begin{align*} f(u) &= u^4\Rightarrow f'(u)=4u^{4-1}=4u^3 \end{align*}


Svar #7
17. september 2018 af rikke0227

Spørgsmålet var nu mere hvordan jeg intergrer 4(1 - x)^3


Brugbart svar (0)

Svar #8
17. september 2018 af guuoo2

Du skal integrere  4u3  
Dvs. du skal finde noget, der differentieret giver 4u3


Brugbart svar (1)

Svar #9
17. september 2018 af SuneChr

# 0
Vil du bringe en hilsen til forfatteren til opgaven og sige, at det hedder integral.
Forfatteren vil sikkert også gerne ha', det skal hedde integrale.
Vi regner kun med
    et integral, integralet, flere integraler, alle integralerne
på Studieportalen.


Brugbart svar (1)

Svar #10
17. september 2018 af mathon

                 \small \int_{0}^{1}\left 4( 1-x \right )^3\mathrm{d}x=-\int_{-1}^{0}4u^3\mathrm{d}u=-\left [ u^4 \right ]_{-1}^{0}=-\left ( 0^4-(-1)^4 \right )=-\left ( -1 \right )=1

         

\small \textup{\textbf{integralregning}}


Svar #11
17. september 2018 af rikke0227

#10

                 \small \int_{0}^{1}\left 4( 1-x \right )^3\mathrm{d}x=-\int_{-1}^{0}4u^3\mathrm{d}u=-\left [ u^4 \right ]_{-1}^{0}=-\left ( 0^4-(-1)^4 \right )=-\left ( -1 \right )=1

         

\small \textup{\textbf{integralregning}}

Som det sidste hvorfor er det at grænserne bliver ændret som de gør i andet trin


Svar #12
17. september 2018 af rikke0227

Altså hvordan får du den nye nedregrænse til -1 ved ikke hvorfor men får den til 1.

u = 1 - x

Unedre = 1  - 0 = 1 (hvordan får du minus)

Medmindre 

Unedre = 0 -1 = -1 (men kan du godt ændre stillingen)


Brugbart svar (1)

Svar #13
17. september 2018 af guuoo2

\small \int_{0}^{1}\left 4( 1-x \right )^3\mathrm{d}x=-\int_{1}^{0}4u^3\mathrm{d}u=-\left [ u^4 \right ]_{1}^{0}=-\left ( 0^4-1^4 \right )=-\left ( -1 \right )=1


Brugbart svar (0)

Svar #14
18. september 2018 af mathon

#11

\small \small \textbf{sammenh\ae ng ved integralregning med substitution:}

    \small \textup{N\aa r F(x) er en stamfunktion til f(x)}
    \small \textup{g\ae lder:}

                            \small \left (F(g(x)) \right ){\, }'=f(g(x))\cdot g{\, }'(x)
    \small \textup{hvoraf:}
                            \small F(g(x))=\int f(g(x))\, \mathrm{d}x
    \small \textup{og}
                            \small \int_{a}^{b}f(g(x))\, \mathrm{d} x=F(g(b))-F(g(a))=F(\beta )-F(\alpha )=\left [ F(u) \right ]_{\alpha =g(a)}^{\beta =g(b)}=\int_{\alpha }^{\beta }f(u)\mathrm{d}u

    \small \textup{med}
                            \small \begin{array}{llccl} &u=g(x)&\mathrm{d}u=g{\, }'(x)\mathrm{d}x&\int_{a}^{b}\rightarrow \int_{\alpha }^{\beta } \end{array}


Skriv et svar til: Intergralregning

Du skal være logget ind, for at skrive et svar til dette spørgsmål. Klik her for at logge ind.
Har du ikke en bruger på Studieportalen.dk? Klik her for at oprette en bruger.